Friday, August 31, 2018

quantum field theory - IR divergence and renormalization scale in dimensional regularization





  • Is it possible that if a certain (loop) integral is IR divergent then that will have effect on the dimensionally regularized answer for that? (..does the epsilon expansion see the IR divergence in any way?..)




  • If a certain epsilon expansion shows double poles (or higher?) then does this have an interpretation? (like is that a sign of IR divergence in the theory?)




I am not necessarily talking of QFT but dimension regularization as a method to regulate integrals in any field theory (may be classical).





  • In dimension regularization is the only way to determine the value of the counterterms is solely through setting of some predefined value at a renormalizaton scale? (then one resubstitutes the counterterm thus calculated into the correlation function being calculated and hence necessarily all evidence of the counterterm is guaranteed to vanish in the final answer?)


    OR




after doing the dimensional regularization is it legitimate to just choose the counter-term such that it removes the 1/epsilon poles? - in this way of thinking one doesn't need to have a renormalization scale. (or after this regularization will the renormalization scale necessarily show up in trying to set the finite/physical/renormalized part of the counterterm?)



Answer



Yes, the infrared divergences are guaranteed to be present in any meaningful computational scheme for a loop diagram, including dimensional regularization, because their existence is an objective question.


In dim reg, UV divergent integrals have to be calculated in $4-\epsilon$ dimensions so we imagine that the dimension is lower than the dimension of interest (or the dimension where the integral is divergent for the first time).


Similarly, IR divergences become worse in lower dimensions, so dim reg means that we imagine that we work in $4+\epsilon$ dimensions, or above the dimension where they get divergent. If both divergences are present, we must separate the integration interval to IR and UV portions and treat them independently, in a dimension that is oppositely moved away from four.


Dim reg should generally lead to simple poles only, $1/\epsilon$. The exponent above $\epsilon$, which is generally $-1$ for all divergent integrals in dim reg, isn't the same exponent that determines the "degree" of the divergence as measured in the momentum space.



The UV and IR divergences are treated analogously in dim reg except that we imagine the dimension to be lowered or raised, respectively. However, their physical interpretations are completely different. UV divergences imply the need to complete the definition of the theory at short distances. IR divergences exist even if the theory is completely well-defined and they encode genuine physics. They seem pathological but this should be blamed on the person who was asking the question. If he asks a more physical question that can actually be measured, the IR divergences cancel. So the presence of IR divergences doesn't mean that we should add counterterms with parameters whose values will matter.


The right treatment of the theory – renormalization – requires the cancellation of the $1/\epsilon$ terms in dim reg: they're analogous to $\log \Lambda$ terms in some cutoff treatments and have the same interpretation. So we're adding counterterms. You say that "all evidence of counterterms vanishes in the final answer" but this is a misleading interpretation. The more correct comment is that "all evidence of counterterms and divergent loop contributions constructed from the original interaction vertices vanishes in the final answer". They cancel. They're equally important. It's a mistake to forget one of them because it would lead you to a completely incorrect belief that the counterterms are unnecessary. They're necessary to cancel the divergences that were there to start with, before counterterms were added. The coefficients of the counterterms have an infinite part that cancels but the finite remainder survives and produces a genuine finite parameter.


Dim reg is a regularization that doesn't introduce any cutoff scale $\Lambda$ – indeed, divergent terms in the limit $\Lambda\to\infty$ are exactly those that we wanted to avoid by using a different scheme – but its results include $1/\epsilon$ terms that are analogous to $\log\Lambda$ in a cutoff approach. The power law divergent diagrams $\Lambda^n$ from a cutoff approach typically lead to convergent integrals in dim reg but their presence in the cutoff approach still implies a sensitivity on the detailed physics at a high-energy scale.


While dim reg has no $\Lambda$, it needs the scale $\mu$ to be added to the integrals over the momentum space for simple dimensional reasons (right units). We don't need to imagine that $\mu$ is as high as $\Lambda$ or that it is a cutoff scale; it may be a scale close to the mass scales where interesting phenomena occur in the theory.


Regardless of the choice of the regularization technique, the right way to think about fine-tuning etc. is to think about the sensitivity of the low-energy physics on the detailed values of the parameters chosen for the parameters at the high-energy scale. This dependence on dynamical parameters in the high-energy theory is also the correct invariant explanation of "what is actually wrong about the UV divergences". It's not the infinities themselves; what's bad is the infinite number of unknown parameters one may choose for the high-scale theory in a non-renormalizable theory.


I believe that the paragraphs above answer (almost?) all the questions by the OP but I chose not to reproduce the precise wording because it sometimes seemed confusing to me, encouraging the answer to be phrased in framework that isn't quite right.


mathematics - Check to see if a Configuration is Possible: prove there's an Hamiltonian path on a connected subset of the square grid graph




Alright, here's a new one for you guys. Instead of solving it, you need to determine if any given configuration is possible to solve, without actually solving it. It needs to be a general method of figuring it out, and should apply to most (if not all) situations.



Without further ado, here's the puzzle you have to use:


Imagine a square grid of arbitrary size. Now picture a T-shape drawn on it; the T is 3 blocks wide and 5 blocks tall as seen in the image below left.


Now picture starting on the top left, and staying within the T shape/boundaries, slowly progressing, filling each square as you go. Can you fill in all the squares of the grid? Of course not! The most you can do us fill 6 squares!


However if the shape itself was a square (like the shape below right), you can fill the entire shape.


T and square shapes on a grid


How to move: Move to any orthogonally adjacent square which you have not already visited in the shape.


Note: For all situations you may assume to start in the upper left corner. Accepted answer goes to the simplest method.




Answer



What you seem to be asking (assuming the squares are completely filled), is to find an Hamiltonian path on a connected subset of the square grid graph.


Defintions (lifted from Wikipedia)



A Hamiltonian path (or traceable path) is a path in an undirected or directed graph that visits each vertex exactly once.


A square grid graph is the graph whose vertices correspond to the points in the plane with integer coordinates, x-coordinates being in the range 1,..., n, y-coordinates being in the range 1,..., m, and two vertices are connected by an edge whenever the corresponding points are at distance 1.



As far as I can tell, the answer is that the Hamiltonian path problem is an NP-complete problem, which means that there is no polynomial time algorithm to determine if an arbitrary graph contains a Hamiltonian path. In other words, given a large enough shape, it might take nearly forever just to find out if there is a solution to the stated problem.




A simple heuristic can determine if there is no solution: If a square is connected to only one other square (i.e. it has a degree of 1), it must either be the first or last square visited, since you can't backtrack. Hence, any shape with more than 2 such squares (marked with "1" in the image below) has no solution.



However, the converse is not true. The shape below right has 2 squares with degree 1, but is unsolvable as well.


vertex degrees added




Another heuristic (courtesy of Lopsy): color the squares like a checkerboard. The number of white and black squares can only differ by 1 if it is solvable. If e.g. there is one more white square than black, then the path must start and end on white.


In this case the T shape satisfies the heuristic while the shape on the right does not. Both are not solvable which again shows that the converse is false.


checkerboard




You can also simplify shapes by removing a vertex of degree 1 if and only if it is connected to a vertex of degree 2. For example, the following shapes are equivalent in terms of their solvability:


equivalent T shapes





Lets put what we have learned to use. Here is a complex shape which satisfies both heuristics (only 1 vertex of degree 1, and there are 7 black squares vs 6 light squares). But we can remove the vertex with degree 1 because it is connected to a vertex of degree 2, and now the new shape does not satisfy the checkerboard heuristic anymore. So, the shape has no hamiltonian path.


Complex shape




This is really a question more for math or computer science:
https://cstheory.stackexchange.com/questions/tagged/hamiltonian-paths
https://mathoverflow.net/questions/tagged/hamiltonian-paths


This may also be of interest (algorithms for finding Hamiltonian paths for L, C, F, and E shaped graphs on a grid): http://www.hindawi.com/journals/jam/2012/475087/


Why isn't this chess puzzle trivial?


This chess puzzle has become popular in the last few days:



enter image description here


But doesn't the white king just need to wander to where the cross is, then move the white pawn forward, then the black bishop takes it and it is a draw in 50 moves.


This question was posed by Roger Penrose who is a great and successful academic. Why it is interesting? Is it really true that computers can't solve it?



Answer



This has something to do with the way computers evaluate positions. They will first count the value of each side's pieces (usually: pawn = 1, knight/bishop = 3, rook = 5, queen = 9) and then some other things, like pawn structure and mobility (number of possible moves). Based on that, Black is much better in this position. Computers don't have a way to 'see' that Black can't liberate himself from the position without White's help, and because the bishops are limited to the dark squares, they can't mate White on their own, or capture the white pawns, liberating the other pieces.


A computer will just 'mindlessly' move the Black bishops around, and only when almost 50 moves have been made, conclude that there is no progress to be made and the game is a draw (because of the 50-move rule, which says that after 50 moves without captures and pawn moves, the game is a draw).


It is said that only if you remove two of the three bishops, some engines are able to see it's a draw. The position as shown is too hard for them; see these two ChessBase articles. The image below, from the article by Sagar Shah, shows that a modern engine thinks Black has an advantage of almost 25 pawns:


enter image description here


Oh, and by the way: the solution you mention actually gives the draw away. The c6 pawn is needed to keep the black king inside his cage. The actual solution is just to move your king along the white squares and nothing else.


Centrifugal force in Arthur C. Clarke's Rendezvous with Rama


I'm currently reading Arthur Clarke's Rendezvous with Rama. Rama is a 50 km long, 16 km diameter cylindrical alien artifact that is rotating fast enough to provide a 0.6G artificial gravity on its long inner surface. The characters in the book enter Rama through an air lock at the hub at one end of the cylinder's central axis, where they are weightless; they then descend a series of ladders and stairs, getting heavier as they go, until they reach full weight at the surface.


One character has a flying contraption with him. He sets off from the hub, weightless, with the intent to ride along the axis the entire length of Rama. He is warned not to drop down towards the surface, as lowering at all will increase the weight (or centrifugal force) he feels. But I'm not sure if this is true.



Centrifugal force is not like gravity, which reaches out to pull you down. If there is nothing pushing him along the rotation, he shouldn't feel any centrifugal force at all, right? Of course, the air itself is going to be moving along with Rama's rotation, but would the air near the hub be moving fast enough to exert a serious sideways force on him, causing him to take part in Rama's rotation? Theoretically, if there was no air, he could lower himself almost all the way to the surface (which from his vantage point would be moving by very fast), and still remain weightless the entire time.


So, am I thinking about this entirely wrong? Or is Clarke?




quantum mechanics - Double slit experiment near event horizon


What happens if you perform a double slit experiment near an event horizon, if one of the slits is outside, one is inside the event horizon?



Answer



This is just a little bit subtle, because at first it looks like nobody has causal access to both light rays, which is required for the existence of interference pattern. On the other hand the principle of equivalence seems to assure us that local experiments done by a freely falling observer would not be affected by the presence of the horizon. There is a sense in which this is true, as follows.


For an observer freely falling into the horizon of a large black hole while conducting the double slit experiment, there will not be any change in the results of the experiment. Gravity is weak and nothing special happens when they cross the horizon, they will only discover they fell into a black hole sometime in the future when tidal forces will make life very uncomfortable. Any experiment localized within space and time will be the same up to tiny corrections due to weak gravity. This is true also if one of the slits (and therefore necessarily the light source) happens to be outside the horizon at the time of the experiment. Of course, for the freely falling observer that statement does not mean much, only the outside observer will be able to make meaningful distinction between inside and outside the horizon.


On the other hand, for an observer staying outside the horizon, one of the slits is invisible, they don't have access to all the light rays, and they will not see any interference. In fact they will not even be aware that there is a double slit experiment going on. There is no problem there because they are not freely falling observers, and there is nothing that states that the result of their experiment is identical to the same experiment made in flat space.


The fact that infalling and outside observers describe the same phenomena so differently lead to many black hole paradoxes, where quantum mechanics and GR (or the principle of equivalence) just barely co-exist without contradiction. Lenny Susskind's popular book on black holes has a good discussion, I think.



Edit: If you jump into the black hole you can have access to the interference pattern after the fact. So, it looks like you can generate another paradox, in addition to the original one. Look at arxiv.org/abs/0808.2096 for the resolution of that one. Roughly speaking, after you jump into the black hole you have only finite time to make measurements before you encounter the singularity. This time is insufficient to get more information than you are entitled to by the rules of quantum mechanics. This statement requires a detailed calculation which is in that paper.


Thursday, August 30, 2018

cipher - Weird messages from a friend


I'm really worried about my friend Friedrich, until today I haven't heard anything from him for the last few weeks. I decided to message him and ask if anything is wrong, and he responded with some weird messages. Now I'm even more concerned! I screenshotted the messages for you to see (I also included a text-only version).


a completely irrelevant image description




Me on Fri, 09/09/2016 18:15:
I noticed you haven't been online for a few weeks. Is everything OK?

I/MFree on Fri, 09/09/2016 18:18:

CEPZH WJAIE APU

Me on Fri, 09/09/2016 18:24:
What the hell are you saying? What happened?

I/MFree on Fri, 09/09/2016 18:27:
CQJXK URREC EOEDK JCPNS TGOTP TPRNF YFBQV CV

Me on Fri, 09/09/2016 18:32:
You're really freaking me out now... Please just answer normally! Where are you?


I/MFree on Fri, 09/09/2016 18:39:
VUFAI ILSZP IYGWV FZHAQ EXPZH GBHEH CGJKN IJFOY KPZTR EUDKJ KCFOZ FYQRC IKUZN ATETJ NAEQL XJLKR J



Some additional clarifications/info:



  • The dates are in the format dd/mm/yyyy.

  • The time is in the format hh:mm.

  • He, for some reason, changed his username to I/MFree.


  • I'll update if anything worth mentioning happens, assuming someone didn't solve it by then.




Update #1



He just sent me a new message! It seems to be independent from the others, since he only responded when I sent something before. Maybe it's not even encrypted!


I/MFree on Sat, 10/09/2016 10:12:
WP-WP-WP RS-RS-RS R-WO-WO-WO

Update #2



I just sent him a message on Sat, 10/09/2016 20:37, but he never responded back... Weird... Before he always responded to every message! And considering where he comes from, he shouldn't be asleep so early!




Update #3



What does the fact that he is not named "Frederick" but "Friedrich" most likely say about him?



Update #4 (two seperate hints)



The spaces in the message of Update #1 may be more important seperators than you think.

Enigma models






What is he trying to say, what happened to him and where is he?



Answer



Piggybacking off of everyone else, the solution is to use



An Enigma Machine, where the timestamps on our messages determine the settings of the machine as follows:

Grundstellung: The letters in the three-letter day abbreviation
Ringstellung: In order, the day, month, and last two digits of year
Umkehrwalze (reflector): Presumably determined by the tens digit in the hour - but this is 1 in all cases and we can just use reflector A
Walzenlage: In order, the ones digit of the hour, the tens digit of the minutes, and the ones digit of the minutes



As an example, our first message




has timestamp Fri, 09/09/2016 18:15 indicating the settings
Reflector A, with wheels VIII I V initially set to 9-9-16 and the display initially set to F-R-I



In order, Friedrich's messages decode to:



NONOT HINGI SOK
SOMEO NEKID NAPPE DMEAN DIMBE INGMO NITOR ED
FOURZ ERODO TSEVE NFIVE NINET WOTWO ZEROC OMMAM INUSO NEONE ONEDO TNINE ZEROO NETHR EEFIV ESEVE N




the last of which are coordinates to Friedrich



40.759220,-111.901357



electromagnetism - Radiated power and energy density for a black-body


I am having an hard time trying to understand why the radiated power per unit area $P$ of a black body is given by $$P=\frac{c}{4} u$$ in terms of the energy density $u$ and the velocity of light.


I know there is a derivation in HyperPhysics, but I did not find it particularly convincing. I cannot understand the physical significance of the parameter $\theta$ and what is its relationship with the the total energy per unit time provided by a unit area. Could someone explain it better to me?



Answer



I think this is just matter of geometry related to the fact that the surface volume of a sphere is four times the apparent cross-section. Imagine yourself between two parallel plates. First consider the energy from one plate. If it radiating parallel beams of light perpendicular to its surface, the energy density would just be P/c. But the "average speed" of energy coming off the surface is not really the speed of light, because the average angle is not 90 degrees. This gives a factor of 2...there's twice as much energy density as the "naive" calculation. But this still isn't the energy density of the black body field, because you're looking at the vaccuum of empty space on one side. That's why you need the second plate. This doubles the energy density again, giving you the factor of four.


EDIT: Okay, I've redone this as carefully as I can, and I'm not getting the factor of four.


I start with a hollow sphere of radius 1 meter and let the temperature be such that the radiated power is 1 watt/m^2. I need to figure out how much energy is inside that sphere. (The naive calculation says it's just volume*power/c.) But this doesn't really work because the energy is criss-crossing in all directions.


I get around this by allowing the shell to instantaneously disintegrate so the energy explodes outwards in a spherical shell. At a sufficiently great distance, the energy is all flowing in the same direction so the naive calculation becomes correct. Letting c=1, this is where I find the energy to be located: enter image description here



The factor of pi/4 on the shell energy comes from the profile of the energy pulse...it actually replicates the cross section of the sphere. When I put the total shell energy back into the volume of the original sphere, I get an energy density of 3*pi/2. The calculation looks right to me but it doesn't line up with either of your sources.


EDIT 2: Okay, I found my mistake! I did some 2-d integration that should have been in 3 dimensions. This diagram explains my corrections (shown in green):


enter image description here


The power which shows up in a given time interval delta-t is the energy which came from the disc as shown. The areas of these discs are proportional to an inverted parabola (area factor 2/3) instead of a half-circle (area factor pi/4). When I make this correction the energy density in the spherical volume comes out to 4, as hoped for.


(I can't explain an additional factor of pi that seems to show up in your Wikipedia reference.)


gravity - How to get "massless" equation of motion from the action of Nordstrom scalar field theory?


There is Nordstrom theory of the particle moving in a scalar field $\varphi (x)$: $$ S = -m\int e^{\varphi (x)}\sqrt{\eta_{\alpha \beta}\frac{dx^{\alpha}}{d \lambda}\frac{dx^{\beta}}{d \lambda}}d\lambda . $$ How to get the equation of motion for massless objects? In this case I may introduce new parameter $\sqrt{\eta_{\alpha \beta}\frac{dx^{\alpha}}{d \lambda}\frac{dx^{\beta}}{d \lambda}}d\lambda = \frac{d\tau }{m}$.


But when I try to derive (like this way) the equations of motion, I get $$ \partial_{\alpha } \varphi = m^{2}\frac{d }{d \tau}\left( u_{\alpha} e^{\varphi}\right)e^{-\varphi}. $$ Unfortunately, this equation doesn't predict the absense of deflection of light. But Nordstrom theory really predicts the deflection of light. So where is the mistake in my reasoning?


Addition.



I got the mistake. By introducing new parameter $\tau$ I must rewrite my equation in a form $$ \partial_{\alpha } \varphi = \frac{1}{m^2}\frac{d }{d \tau}\left( p_{\alpha} e^{\varphi}\right)e^{-\varphi}. $$ So it really preficts no deflection of light.




homework and exercises - How to calculate Riemann and Ricci tensors for a sphere?



Let's have the metric for a sphere: $$ dl^{2} = R^{2}\left(d\psi ^{2} + sin^{2}(\psi )(d \theta ^{2} + sin^{2}(\theta ) d \varphi^{2})\right). $$ I tried to calculate Riemann or Ricci tensor's components, but I got problems with it.


The Ricci curvature must be $$ R_{ij}=\frac{2}{R^{2}}g_{ij}. $$ But when I use definition of Ricci tensor, I can't turn the expression into the expression for the metric tensor


Maybe, there are siome hints, which can help?




riddle - Chewed, Wielded and Legendary - What am I?


    I have teeth, but can be chewed.


    I alone am worshipped, but those who wield me are too.


    I reside among legends, but a myth I am not.


What am I?


Edit: Huge apologies as I was using yield and not wield. I think that may have been what was throwing some people off.


Hint:



The one you would bite is not physically the same as the one you would wield.




Hint 2:



The concept of wielding is important - don't disregard it as other answers have.



Hint 3:



In the past I was worshiped symbolically not physically - but not in modern day




Answer




Second try since the 'yield' line was changed to 'wield':


You are searching for a...



Trident



I have teeth, but can be chewed.



The three spikes are the teeth, Trident chewing gum



I alone am worshipped, but those who wield me are too.




Its a precious weapon. And the god Poseidon is whorshipped.



I reside among legends, but a myth I am not.



Well a god might be a legend, but the item sure isn't



 
 
My first try, OLD ANSWER: I go with:




GOLD



I have teeth,



Gold grillz



but can be chewed.



Gold is a kinda easy to bend metal. So some people bite it, to see, if it's real Athletes chewing gold medal




I alone am worshipped, but those who yield me are too.



Gold digger



I reside among legends, but a myth I am not.



Funerary goods. Like in a Pharaoh's grave.



quantum mechanics - Does simply putting a photon-splitting crystal after a double slit break the interference pattern?


A modified version of the quantum eraser experiment, consisting of just a photon source, double slit, photon-splitter crystal and a screen, capturing one of the entangled twins. No detectors, no mirrors, nothing. The other twin is lost. Would this produce an interference pattern or not? The sources I find are not clear on this.


Here is the impliaction: If this modified experiment produces an interference pattern and the full quantum eraser experiment does not (when placed in the mode which unambiguously determines which slit the photon passed through, so the "eraser" part is missing), it means we can send information to the past. Imagine this: I send photons in a wave, split them, using the crystal and look for patterns on the screen, produced by the screen-side twin photon. The other twin I bounce between mirrors until I want to send the information to the past. Lets say the format of the information is binary - "0" encoded by NOT producing an interference pattern and "1" by producing an interference pattern. So if I want to send "0" I will add the detectors to the setup (resulting in the classic setup, with the eraser missing), and if I want to send "1" I will remove them (resulting in my modified setup).



This all hinges on the answer of the title: "Does simply putting a photon-splitting crystal after a double slit break the interference pattern?"


EDIT: As requested here is what I imagine and ask if the photons, hitting the screen (D0) will produce an interference pattern.The proposed setup



Answer



Here is a drawing of what happens before and after the crystal that can give spontaneous parametric down conversion, SPDC:


onetotwophotons



An SPDC scheme with the Type I output


The conversion efficiency of SPDC is typically very low, with the highest efficiency obtained is on the order of 4 pairs per $10^6$ incoming photons for PPLN in waveguides. However, if one half of the pair (the "signal") is detected at any time then its partner (the "idler") is known to be present.



This is a "crystal+photon" interaction, that allows for the two produced photons to be entangled. Note the term interaction.



The incoming photon in the double slit experiment comes with a specific wave function, which is the solution of the quantum mechanical problem "photon hitting two slits of specific distance apart with specific dimensions", and thus one can get the interference pattern one sees in single photon at a time experiments, as this in this one here..


Any interaction after passing the slits will change this wavefunction, and due to the probabilistic nature of quantum mechanics the original phases in the wavefunction of the incoming photons, which carry the memory of the double slits, will be lost whether interacting with an atom in a detector or with a whole crystal splitting into two.


This is seen clearly in this which way experiment with electrons:



“When the electron suffers inelastic scattering, it is localized; this means that its wavefunction collapses and after the measurement act, it propagates roughly as a spherical wave from the region of interaction, with no phase relation at all with other elastically or inelastically scattered electrons,” Frabboni said. “The experimental results show electrons through two slits (so two bright lines in the image when elastic and inelastic scattered electrons are collected) with negligible interference effects in the one-slit Fraunhofer diffraction pattern formed with elastic electrons.”



In the image in this answer, the photon scatters inelastically on the total crystal.


It is enough to interact, for the double slit interference pattern to be lost.


I cannot comment on the implications you wanted, but can state that the original incoming interference pattern (phases) are lost.


Wednesday, August 29, 2018

thermodynamics - Was Joule's experiment able to show: thermal energy = $mgh$


Is my understanding of this experiment correct? There is the quantity $MC\delta \theta$ associated to the heat which I think is not an absolute value to be compared with $mgh$ since the $\delta \theta$ could be defined arbitrarily. So this experiment does not prove : thermal energy = $mgh$ it could however show a linear relation between $\delta \theta$ and $h$.





mathematics - A Fairly Simple Riddle



A homeless man who lives on the streets has a smoking problem. However, since he can not buy cigarettes, he is forced to collect cigarette butts. He needs six butts to make a single cigarette. If he collects 216 butts, how many cigarettes can he make?



Hint: After making six cigarettes and smoking them, he can use those butts to make another one.






classical mechanics - Why does a ping pong ball change direction when I spin it on a table?


When I spin a ping pong ball on the table, it rolls forward in the opposite direction of the spin, and then eventually changes direction and rolls backward.


Here's a video demonstrating the effect.


Why does that happen?



Answer



One way to explain it that makes sense to me is that the backwards spin on the ball is a force pushing the ball toward you. The one time applied force of your finger that creates the backward spin also pushes the ball forward, but the backspin stays almost constant. At the moment you spin the ball, the forward force is greater then that of the backspin, so the ball moves away. After time zero, the backspin is the only force acting horizontally (you aren't still touching the ball, so there is no applied force) and so the ball slows, then returns because of a negative acceleration.


If you were to graph this motion as Position (away from you) vs Time, you would get a graph that looks similar to the motion of a ball thrown upwards and pulled back by gravity.


newtonian mechanics - Is centripetal acceleration mandatory for circular motion?


If we consider a case where a closed circular wall is present, and inside the boundary, just adjacent to the inner wall, two particles are placed touching the surface of inner boundary such that the distance between both particles tends to 0. Particle A and B have identical masses, shapes and physical properties. Particle B applies a constant force on particle A and particle B is in uniform circular motion. Thus, particle A also moves in a circle. And here, centripetal force is not required to make particle A move in a circle.


What's going on?



Answer



Let me try to add a missing intuitive piece in the explanation to the discussion to @Gert's answer of why there must always be radial acceleration for any circular motion.



Remember what acceleration is: Change in velocity. $\vec a=d\vec v/dt$. More mathematically it is change in the velocity vector.


If velocity is changed - either magnetude or direction - then we define this rate of change as acceleration.



  • If the velocity magnetude is changed, then acceleration is pointing in the same direction as the velocity (and it goes faster or slower). We call this tangential acceleration.

  • If the velocity direction is changed, then acceleration is pointing to the side and not parallel to the velocity direction. If it is exactly perpendicular to the direction we call it radial acceleration.


Now imagine what happens if an object has no tangential and only radial acceleration. Then it keeps its magnitude (so the speed doesn't change) and only changes direction. Thinking more about this it should be clear that such a situation is a circular motion. If there both is tangential and radial acceleration, then the we have an elliptical motion.


Naturally, for any path that is not straight, there must be a change of the velocity direction. And we call such a change acceleration - for a circular motion that acceleration happens to be perpendicular and towards the center of the circular path.


Any forces that act on a particle in a circular motion therefor must result in such a sideways acceleration. If they do not, then the motion cannot be circular because of the above explanation.


classical mechanics - Why the Principle of Least Action?


I'll be generous and say it might be reasonable to assume that nature would tend to minimize, or maybe even maximize, the integral over time of $T-V$. Okay, fine. You write down the action functional, require that it be a minimum (or maximum), and arrive at the Euler-Lagrange equations. Great. But now you want these Euler-Lagrange equations to not just be derivable from the Principle of Least Action, but you want it to be equivalent to the Principle of Least Action. After thinking about it for awhile, you realize that this implies that the Principle of Least Action isn't really the Principle of Least Action at all: it's the "Principle of Stationary Action". Maybe this is just me, but as generous as I may be, I will not grant you that it is "natural" to assume that nature tends to choose the path that is stationary point of the action functional. Not to mention, it isn't even obvious that there is such a path, or if there is one, that it is unique.



But the problems don't stop there. Even if you grant the "Principle of Stationary Action" as fundamentally and universally true, you realize that not all the equations of motions that you would like to have are derivable from this if you restrict yourself to a Lagrangian of the form $T-V$. As far as I can tell, from here it's a matter of playing around until you get a Lagrangian that produces the equations of motion you want.


From my (perhaps naive point of view), there is nothing at all particularly natural (although I will admit, it is quite useful) about the formulation of classical mechanics this way. Of course, this wouldn't be such a big deal if these classical ideas stayed with the classical physics, but these ideas are absolutely fundamental to how we think about things as modern as quantum field theory.


Could someone please convince me that there is something natural about the choice of the Lagrangian formulation of classical mechanics (I don't mean in comparison with the Hamiltonian formulation; I mean period), and in fact, that it is so natural that we would not even dare abandon these ideas?




Tuesday, August 28, 2018

logical deduction - Water island puzzle


On a planet called T, there exists a continent called P, which is surrounded on all sides by water. The continent comprises eight countries – A, R, B, C, D, E, G and K – located contiguously on it, i.e., the total area of the eight countries is the same as the area of the continent.


Each country is exactly in the shape of a square, with the four edges of the square as its boundaries. The areas of the eight countries are all equal. Any country is said to be a neighbour of another country if the two countries have one edge as a common boundary.


Further, one can travel by land between two countries only if the two countries are neighbours and it is known that one can reach any country from any of the other countries by travelling by land (passing through one or more countries, if required).


It is also known that




  • R and E are the only neighbours of D, while K is the only neighbour of B, but K is not to the west of B.

  • A and D are the only neighbours of E, while K is the only neighbour of C.

  • E is directly to the north of R.

  • G is directly to the east of D and no country is present directly to the south of G.


Source: time


What is the approach for this puzzle? Q:



Which of the following countries is to the immediate South of K?
a) B

b) C
c) G
d) No country is to the immediate South of K.




Answer



I have the same solution as already posted. Without following its reasoning, here is mine:



Quickly establish that E - D - R must run north to south. Now G is somewhere east of D but not adjacent


 
E

|
D G
|
R

G must be approached from the north.
Only A or K can touch G.
Only A or K can touch E.
Suppose it is E - K - A - G
 

E - K - A
| |
D G
|
R

Now B and C must touch K but one of them would have to join D and G.
But both B and C only have one neighbour K.
So it must be E - A - K - G and as K is not to the west of B it must be:
 

B
|
E - A - K - C
| |
D G
|
R

So the answer is




c) G is to the immediate South of K.



word - What in the shells is this puzzle?



Clergyman's wife tells story of picking up a muscle.




Solution: $\require{color}\definecolor{hint}{RGB}{1,124,233} \small\color{hint}{\textsf{5 words}}$



Answer



From the title:



The word that jumps out the most is "shells". My first thought was that this might mean that we need to find synonyms and only take the outer/inner letters, but this didn't lead me anywhere. Later in The Sphinx's Lair @Alconja suggested that maybe "shells" means the words could be arranged in a matryoshka doll-like pattern, i.e. A(B(C(D)E)F)G, which I thought was a really good idea.



From the hint:



The color of "5 words" is 0x17ce9 = 97513, which I think gives the length of each of our 5 words in order.




Putting this together:



Clergyman's wife tells story of picking up a muscle.
a

Clergyman's wife tells story of picking up a muscle.
lat

Clergyman's wife tells story of picking up a muscle.
elate

Clergyman's wife tells story of picking up a muscle.
relates

Clergyman's wife tells story of picking up a muscle.
prelatess




geometry - 20 right isosceles triangles into a square


Similar: Unlucky tiling: Arrange thirteen right isosceles triangles into a square


Five graded difficulty isosceles right triangle into square tilings


Two difficult "Seventeen right isosceles triangles into a square" tilings


The challenge is to fit the smallest $20$ integer-sided scaled right isosceles triangles into a square with diagonal $46$


For convenience, I list the areas of the triangles:


$1, 2, 4, 8, 9, 16, 18, 25, 32, 36, 49, 50, 64, 72, 81, 98, 100, 121, 128, 144$


There are four ways of doing it (not two as originally posted). A brave person it would be who tackled this by hand.


By way of illustration/clarification, here are the right isosceles triangles of area


$1, 2, 4, 9, 16, 18, 50$



arranged into a $10\times 10$ square:


10x10_7



Answer



Here are at least two solutions (up to reflection and rotation)



enter image description here
The trapezoid outlined in red can be flipped.
enter image description here
This is presumably the other "substantially different" solution (not counting the above trapezoid flip as different).




mathematics - Find the value of $bigstar$: Puzzle 3 - Substitution


This puzzle replaces all numbers with other symbols.


Your job, as the title suggests, is to find what number fits in the place of $\bigstar$.


All symbols abide by the following rules:



  1. Each symbol represents integers and only integers. This means fractions and irrational numbers like $\sqrt2$ are not allowed. However, zero and negative numbers are allowed.


  2. Each symbol represents a unique number. This means that for any two symbols $\alpha$ and $\beta$ which are in the same puzzle, $\alpha\neq\beta$.

  3. The following equations are satisfied (this is the heart of the puzzle):



$$ \text{I. }\alpha+\beta=\gamma\times\delta \\ \space \\ \text{II. }\gamma+\varepsilon=\delta \\ \space \\ \text{III. }\gamma-\varepsilon\times\varepsilon=\beta \\ \space \\ \text{IV. }\varepsilon\times\delta+\gamma=\alpha \\ \space \\ \text{V. }\zeta\times\zeta=\zeta \\ \space \\ \text{VI. }\zeta+\gamma=\varepsilon \\ \space \\ \text{VII. }\alpha-\delta-\beta=\bigstar $$



What is a Solution?


A solution is an integer value for $\bigstar$, such that, for the group of symbols in the puzzle $S_1$ there exists a one-to-one function $f:S_1\to\Bbb Z$ which, after replacing all provided symbols using this function, satisfies all given equations.


What is a Correct Answer?


An answer is considered correct if you can prove that a certain value for $\bigstar$ is a solution. This can be done easily by getting a function from every symbol in the puzzle to the correct integers (that is, find an example for $f:S_1\to\Bbb Z$).



An answer will be accepted if it is the first correct answer to also prove that the solution is the only solution. In other words, there is no other possible value for $\bigstar$.


Good luck!


Previous puzzles in the series:


Puzzle 1 Puzzle 2


Next Puzzle



Answer



From V and VI, we have that



$\zeta = 1$. Eq. V implies that $\zeta^2 - \zeta = \zeta(\zeta - 1) = 0$, which means $\zeta = 0$ or $\zeta = 1$. But if $\zeta = 0$, then $\gamma = \epsilon$, which is forbidden.




These then imply, from II, III, and VI, that



$\varepsilon = \gamma + 1$, $\delta = 2 \gamma + 1$, and $\beta = \gamma - (\gamma + 1)^2$.



From I and IV we have that



$\alpha = \gamma + (2 \gamma + 1)(\gamma + 1) = 2 \gamma^2 + 4 \gamma + 1$, and $\alpha = \gamma (2 \gamma + 1) - \gamma + (\gamma + 1)^2 = 3 \gamma^2 + 2 \gamma + 1$. Equating these two, we have $\gamma^2 - 2 \gamma = 0$, implying that $\gamma = 2$ or $\gamma = 0$. But if $\gamma = 0$, then by II $\epsilon = \delta$. Thus, $\gamma = 2$.



From here, we can plug things in:




$ \alpha = 2 + (5\times 3) = 17$ from above; $\beta = 2 - 3^2 = -7$; $\delta = 5$; and thus $\bigstar = 19$.



Monday, August 27, 2018

mathematics - Controlling a robot blindfolded on a 9x9 grid


A robot is located somewhere inside a 9x9 grid shown below, but you don't know where it is. You can send commands to the robot to make it move one cell left, right, up or down. Shaded areas and edges of the grid are walls that cannot be traversed. If a robot hits a wall it stays in its current cell. Your task is to guide the robot to the target cell (shown as T). Once the robot reaches the target, the game will terminate immediately. What is the fewest number of commands you need to send to the robot to guarantee that it reaches the target cell?


This is my favourite puzzle that I have created. Good luck!


enter image description here



Answer



My first attempt was done by hand. It used 28 commands:



RRR UUU R DDD RR UUU RR LLL UUUUU RRD




but this was not optimal. I have now done a computer search to find optimal solutions.


It found 180 solutions of length



24 commands



No shorter solutions exist.


I'll illustrate the following solution:



LU LU LU UU RRR UU L DDD RRRRRUU




To see how it works, start with a robot on every available square, and send the commands. The robots should all reach the target square at some point.




o o o o x o o o o
o o o o x o o t o
o o o o x o o o o
o o o o o o o o o
x x x o o o x x x
o o o o o o o o o
o o o o x o o o o

o o o o x o o o o
o o o o x o o o o

LU

o o o . x o o o .
o o o . x o o t .
o o o o x o o o .
. . . o o . . . .
x x x o o o x x x

o o o . . o o o .
o o o . x o o o .
o o o . x o o o .
. . . . x . . . .

LU

o o . . x o o . .
o o o . x o o t .
. . o o x . . . .

. . . o o . . . .
x x x . o o x x x
o o . . . o o . .
o o . . x o o . .
. . . . x . . . .
. . . . x . . . .

LU

o o . . x o . . .

. o o . x . . t .
. . o o x . . . .
. . . o o . . . .
x x x . o o x x x
o . . . . o . . .
. . . . x . . . .
. . . . x . . . .
. . . . x . . . .

UU


o o o o x o . . .
. . . o x . . t .
. . . . x o . . .
. . . . o o . . .
x x x . . . x x x
o . . . . . . . .
. . . . x . . . .
. . . . x . . . .
. . . . x . . . .


RRR

. . . o x . . . o
. . . o x . . t .
. . . . x . . . o
. . . . . . . o o
x x x . . . x x x
. . . o . . . . .
. . . . x . . . .

. . . . x . . . .
. . . . x . . . .

UU

. . . o x . . . o
. . . . x . . t o
. . . . x . . . .
. . . o . . . . .
x x x . . . x x x

. . . . . . . . .
. . . . x . . . .
. . . . x . . . .
. . . . x . . . .

L
. . o . x . . o .
. . . . x . . t .
. . . . x . . . .
. . o . . . . . .

x x x . . . x x x
. . . . . . . . .
. . . . x . . . .
. . . . x . . . .
. . . . x . . . .

DDD

. . . . x . . . .
. . . . x . . t .

. . . . x . . . .
. . o . . . . . .
x x x . . . x x x
. . . . . . . . .
. . . . x . . . .
. . . . x . . . .
. . . . x . . . .

RRRRRUU


My computer program used a technique called iterative deepening. Essentially it first tried all command sequences of length 20, didn't find any solutions, then tried all of lenth 21, then length 22, etc. For each length it tried all sequences, essentially a depth first search, but backtracked as soon as a simple heuristic showed that the current position could not be done in the number of moves remaining.


The heuristic I used was simply to count how many U/D/L/R moves were needed for each remaining robot to get to the target, for each of the four move types take the maximum number of times it was used by any robot, and then add those four maxima together.


thermodynamics - Why does ice melt, wait for 100 degrees and THEN vaporise? Why is not the process of expansion of things continuous?



What I am asking is this: Why can't a body be solid, then solid-ish, then solid-like, then liquid-like, then liquid-ish, then liquid, then vapor-like and then vapor? Why is there a rigid temperature boundaries between solid, liquid and vapor? Why doesn't water simply change "states" in a continuous manner?




reflection - Why is reflected light polarised?



Why is reflected light polarised?


I have learnt about Brewster's angle, and how at a particular angle all light reflected is polarised, but do not understand why. Is this something that could be explained to a guy that doesn't have a Ph.D in physics?




Sunday, August 26, 2018

homework and exercises - Constant of gravity in earth fixed coordinate system


I have this problem:



If the constant of gravity is measured to be $g_0$ in an earth fixed coordinate system, what is the difference $g-g_0$ where $g$ is the real constant of gravity as measured when the earth stands still.



The earth fixed coordinate system is rotating, so that the absolute acceleration $g$ is given by


$g=g_0+2\vec\omega\times\frac{d}{dt}\vec r+\vec\omega\times (\vec \omega\times \vec r)$


This is the Coriolis term and the Centrifugal term, the other ones disappear since earth rotates in a steady pace and does not itself accelerate.


I know the answer is supposed to be



$g-g_0\approx g_0\frac{x^2-2x}{2}\cos^2(\phi)$


Where $x=\frac{R \omega^2}{g_0}$. How can I come to this conclusion? My attempts so far have been to try to evaluate the expression given earlier. But I cannot for my life figure out how that yields the supposed answer.


(Note: I've edited the "correct" answer, multiplying what was written originally with g_0 after some of the answers were written)



Answer



Let's assume that we are considering the effective acceleration due to gravity felt by an object at rest relative the the Earth's surface at lattitude $\phi$. Since the object is at rest, $\dot{\vec r} = 0$, and the Coriolis term vanishes.


With the $z$-axis along the axis of rotation, we can write $$ \vec\omega = \omega \hat z $$ On the other hand, the position vector is given by $\vec r = R\hat r$. It follows that the centrifugal force is $$ \vec F_\mathrm{cf} =-\vec\omega\times(\vec\omega\times \vec r) = -\omega^2R \,\hat z\times(\hat z \times \hat r) =-\omega^2 R\,((\hat z \cdot\hat r)\hat z - \hat r) $$ Since it is the radial component of the centrifugal term contributes to the component of the effective acceleration due to gravity perpendicular to the surface of the Earth, we take the dot product of this expression with $\hat r$ to obtain this component; $$ \vec F_\mathrm{cf}\cdot \hat r = \omega^2 R(1-\cos^2\theta) = \omega^2r\sin^2\theta $$ where $\theta$ is the spherical polar angle measured from the positive $z$ axis. Since the polar angle $\theta$ and lattitude $\phi$ are related by $\theta = \pi/2-\phi$, this result can also be written as $$ \vec F_\mathrm{cf}\cdot \hat r = \omega^2 R\cos^2\phi $$ The perpendicular component of the effective acceleration due to gravity is therefore $$ g_0 = g-\omega^2 R\cos^2\phi $$ I'm a bit baffled by the expression you say one is supposed to get. For one thing, the dimensions don't seem to work out since $x$ is dimensionless, but is being equated to something with dimensions of acceleration. However, if one were to take the result I just derived, then notice that it implies $$ \frac{g-g_0}{g} = \frac{\omega^2 R \cos^2\phi}{g_0+\omega^2R\cos^2\phi} = \frac{x \cos^2\phi}{1+x\cos^2\phi} \approx (x-x^2\cos^2\phi)\cos^2\phi $$ where in the last equality we have assumed that $x$ is small, namely that the spin of the earth is slow. This is as close as I can seem to get to the desired expression.


In the delayed choice quantum eraser, what would happen if we were to send the second entangled particle into a blackhole?


In the delayed choice quantum eraser experiment, what would happen if we were to send the second entangled particle into a blackhole instead of the eraser apparatus?



Answer



I wrote up a lot of this stuff a while ago on my github, if it helps -- not specifically what happens if we black-hole the second qubit, but all of the math you'll need to understand the basic results up through the delayed choice quantum eraser.


When you look at your quantum-eraser double-slit experiment, you scan a photon-counter across some $x$-axis and count how many photons you get at each individual point; if you were to just plot the photons coming into any given position place you would see the following red curve: A red bell curve is shown to be a sum of two smaller bell curves, a blue one slightly to the left and a green one slightly to the right.


You can see that this curve is a sum of the blue and green curves; this is additional information that you get by correlating the individual photons you are detecting with a separate counter: the counter of the entangled photon, before you "erase" the interference. This photon is able to tell you "which way" the photon went. In practice we do this by "coincidence counting", we use a polaroid filter, and then use a photon-counter to detect the entangled photon. We then are very careful to only count when both of these two counters fire within some short time interval; ideally that time interval is much shorter than the mean time between counts so that this "coincidence" really means that the detected photons were part of the same entangled pair. Turn the polaroid filter one way and you will see the blue curve; turn it 90 degrees and you will see the green curve. The question of how you turn the polaroid filter is the "choice" you make; for the "delayed choice" quantum eraser you insert an optical delay in the entangled photon before it hits the polaroid filter, and you choose the orientation of the polaroid after the electrical signal for the photon has already been detected.


What's interesting about all of this is if you orient the polaroid 45 degrees one way or the other relative to those two pieces of "which way" information, you lose any way of getting that "which way" information, and the bell curve decomposes into two other curves:



The same red curve is now shown to be the sum of two very wavy curves, green and blue. The blue one is peaked in the center and then the green curve peaks beside it; they both cross up-and-down several times while remaining in a bell-curve-looking


This blue curve that you see is exactly what you would have seen from a double-slit experiment without any of the added "quantum eraser" setup; it's the wavy interference pattern that we're used to. The green curve is also interesting, it is what you would see if you attached two opposite half-wave plates in front of the slits to get them to be 180 degrees out-of-phase. But, you see that these two sum up to the exact same red curve.


If you throw away the photon by any means -- throwing it into a black hole is only one way to do this -- then you have no way to get any of these sub-curves and you'll instead see the red bell curve, not any of these green or blue curves.


thermodynamics - Does wrapping a wet paper towel around a glass bottle really speed up the cooling process?


There are claims like this one that you can improve the cooling speed of beverages when you put them wrapped in a wet paper towel inside the refrigerator/freezer.


enter image description here


I've just tried it by myself and it seems it does not work as expected, although I might have confused freezer with refrigerator.


My question:


Is this kind of an urban legend or does it actually help?



Answer



It may actually work, as evaporating liquids need heat to evaporate, and water will somewhat evaporate even in the fridge.


I am not sure it works in practice, because the paper also causes an adverse effect, it provides insulation,



Hard to tell which effect is dominant.


I'm pretty sure that the balance of both effects depends in a very large part on the physical structure of the paper used.
There are multiple independent physical effects in relation to the paper:




  • On the picture, we see there is air under or inside of the paper in some locations. I would think of the contribution to heat flow to be similar to the cooling from evaporation, except for the sign.
    This is related to the paper structure by depending on how "soft" the paper is - when it is wet, but was not wet for a long time.




  • the total effect of cooling is related to the time the surface of the paper stays wet, and after that, to the time the paper stays wet at all. The mention of a paper towel supports this idea, because paper towels are specifically optimized to keep water.





  • The inner surface area of the paper gets very relevant at the time the surface is not longer "fully wet".




  • For the relevance of the size of the paper's pores, the same holds like for the inner surface above.




  • If the water on and in paper freezes, the result is a layer of - depending on how wet it was - porous material, insolating relatively well.





  • Dry paper is insolating.





There are also relevant effects not related to the paper:




  • The air temperature influences the rate of evaporation. When it is colder, the evaporation is slower, so that the cooling effect is weaker when it's cold.





  • Evaporation depends on humidity of air. And humidity of air in a fridge largely depends on condensation on the surface of the cooling elements. That depends on the surface temperature, which in turn depends both on the user set temperature, but, to a large part on the geometry of the cooling element! [1] But, quite counter-intuitively, not directly on the fridges overall temperature as set by the user.




  • If it's very cold, the upper layer (or more) of the wet paper could freeze, leaving just a little sublimation instead of evaporation for cooling. See the paper section for paper related effects of freezing.




  • Some fridges use an internal fan to make move more cold air to goods hard to reach from the cooling element. As for the bottle moving more air around also means moving more moisture away, allowing for more evaporation, it could help the cooling effect a lot. Additionally, condensation of the moisture elsewhere in the fridge is made quicker in a similar/inverse way.







[1] Simplified, the geometry of the cooling element is about the area available to exchange heat. If it is large, a small temparature difference is enough for cooling. That reduces condensation on the element - keeping more moisture in the air. If it is small, it needs to be freezing cold, collecting moisture as ice. A medium size collects moisture by condensing to water - which is often removed through a small drain pipe. (Simplifying by ignoring the influence on convection near the cooling surface.)

quantum mechanics - How can we describe the electrons of multi-electron atoms (i.e. not Hydrogen) when equations/analytic solutions only exist for Hydrogen?


I've been digging into emission spectra of different elements and found that such things as the Rydberg equation, Bohr's model, and quantum mechanics can only fully describe the single electron in the Hydrogen atom. How did we then make the leap to s,p,d,f shells of multi-electron atoms? How accurate is our analysis of these more complicated elements?


Rydberg Equation (side-note: Is this an empirical 'data-fitting' equation? What's the significance of that?)


$$\frac{1}{\lambda}=R_H\left( \frac{1}{n_1^2}-\frac{1}{n_2^2}\right)$$


Hydrogen: enter image description here


Helium: enter image description here


Iron: enter image description here



Potassium: enter image description here



Answer



The only atoms for which the Schrodinger equation has an analytic solution are the one electron atoms i.e. H, He$^+$, Li$^{2+}$ and so on. That's because with more than one electron the forces between electrons make the equation too hard to solve analytically. However, over the 90 or so years since Schrodinger proposed his equation a vast array of numerical methods for solving it have been developed, and of course modern computers are so powerful they can calculate the (electronic) structure of any atom with ease. This applies even to heavy atoms where relativistic effects need to be taken into account.


The Rydberg equation is an approximation because it does not take the electronic fine structure into account. However it's a pretty good approximation. It works because for a one electron atom the energy of the orbitals (ignoring fine structure) is proportional to 1/$n^2$, where $n = 1$ is the lowest energy orbital, $n = 2$ is the second lowest and so on.


Saturday, August 25, 2018

soft question - Does "finite" include zero?



Infinity, is clearly not finite. But there is some dissent on whether or not zero is finite. I have seen authors use "finite" to indicate the value of $0$ is excluded as well as infinity.


Is there any rule about this?



Answer



There is no rule, and it depends on the context.




  • If you're worried about things being very big, then zero is an OK value to have, and you'd count it as a finite quantity.




  • In other situations, however, you are concerned about whether a quantity $q$ is exactly zero, or whether it is only a finite-precision approximation of it. Thus, you might say that "$q$ is finitely small", but after a while you end up simply saying "$q$ is finite" for that assertion.





It is open to discussion whether this is mathematically correct or a useful shorthand, but the fact is that people do use it and their communications are perfectly clear, so the discussion is pretty moot. Whether zero is 'finite' or not in a given paper should be obvious from the context, and if it is not then the charge against the author should be "ambiguous wording" instead of "incorrect notation".


general relativity - In realistic gravitational collapse, can we have an absolute horizon without a trapped surface?


In gravitational collapse, it seems that there is no close or simple logical relationship between the formation of an event horizon (absolute horizon) and formation of a trapped surface (which implies an apparent horizon).


Modeling gravitational collapse is a specialized and highly technical field, and I don't know much about it. IIRC, simulations can most easily detect the formation of an apparent horizon, whereas the formation of an absolute horizon may not even be easy to pick out, since it's a global notion.



If we observe an object that has formed an event horizon, then I automatically imagine that it has also formed a trapped surface, and therefore must have a singularity due to the Penrose singularity theorem. But what basis do we have for this implication? Is it just a fact that arises from simulations, or is there some theorem that guarantees it?


Question: In realistic gravitational collapse, does the formation of an absolute horizon imply a trapped surface? If there are exceptions, are there strong reasons to believe these are unphysical or not generic?


The following, from Wald and Iyer, doi:10.1103/physrevd.44.r3719 , seems relevant:



...no general theorems require the presence of trapped surfaces in the collapse to a black hole. (The event horizon of the black hole must "settle down" to an outer marginally trapped surface at late times, but will normally have a positive expansion at any finite time. ) Nevertheless, the usual physical arguments concerning why black holes rather than naked singularities should be formed by collapse strongly suggests that outer trapped surfaces always should accompany black-hole formation.



From Hawking and Ellis, p. 321, it looks like "marginally" means the expansion scalar $\theta=0$. If I'm understanding the definition on p. 319 correctly, then "outer" means $\theta\ge0$. Their figure 59 on p. 321 shows an example of astrophysical collapse in which the apparent horizon forms later than the event horizon.


I'm not clear on how to interpret this and would appreciate further explanation. The gist of it seems to be that there are strong reasons to expect a trapped surface in realistic cases. The sentence in parens seems to be saying that due to no-hair theorems, the exterior spacetime has to approach a Kerr-Newman spacetime. I guess this would lead us to expect that the interior would also approach Kerr-Newman spacetime, but maybe this is not an absolute implication. Would the exception be unphysical, e.g., it has to be non-generic?


The final sentence from the Wald paper seems to be making a link with cosmic censorship, but that seems vague to me. Weak cosmic censorship just says there's an absolute horizon, but doesn't say there's a trapped surface.




electromagnetism - Does bunching reduce synchrotron radiation?


A continuous charge distribution flowing as a constant current in a closed loop doesn't radiate. Is it therefore true that as you increase the number of proton bunches in the LHC, while keeping the total charge constant, the synchrotron radiation decreases?



Answer



Synchrotron radiation can be coherent and incoherent. Coherent SR arises when electrons are grouped into short bunches so that the entire bunch emits SR as a whole. Quantum mechanically, in coherent SR the photon emission from different electrons in a bunch sum up at the amplitudes level and constructively interfere. In the incoherent SR they sum up at the level of intensity, and there is no interference.


Incoherent SR does not care how electrons are distributed along the ring, while the coherent SR is obviously boosted up in the presence of strong bunching. So, the more homogeneously you distribute the electrons, the less the effect of coherent SR will be and the less overall SR you'll have.



Now let's look at the incoherent SR. Theoretically, you are right: if we managed to create the absolute homogeneous charge distribution along the ring, we would (classically) have no SR at all because charge distribution does not change in time. The point is that this is not feasible experimentally, at least for the accelerators and the beams we have. That would require putting electrons in a well-defined quantum state of the radial motion and a well-defined angular quantum number m for the azimuthal dependence, and the accelerator technology is very far from that.


However, there is another thing which mimics that situation closely. People have managed recently to put freely propagating electrons in states with well-defined orbital angular momentum (m as high as 75), see this paper in Science for details, and they really see the annular distribution for the electron density. For such a state there exists a reference frame where the electron does not move along the z axis but just rotated as a whole in the transverse plane (with some radial distribution) around the symmetry axis. This rotation is not driven by any force, it's just the peculiar superposition of plane waves that creates this steady pattern. So in this case you can say that the electron indeed circulates but does not emit any SR.


classical mechanics - heap of pebbles


It is common to see a heap of conical shape formed by a large number of similar size hard spherical objects, for example, a heap of pebbles, sand etc. Suppose we want to model this system as a collection of identical hard balls, with dry friction between the balls described by coefficient $\mu_1$ and the friction between the balls and the floor described by coefficient $\mu_2$. Based on a "toy" problem with three cylinders forming a pyramid one can conjecture that there is some threshold condition for $\mu_1$ and $\mu_2$ that guarantees equilibrium for a heap. What are the conditions for a heap of identical hard spheres to be in a static equilibrium?




general relativity - How does a rotating object cause frame dragging?


Frame dragging is a consequence of general relativity.


But I don't really understand it. Of course I can find metaphors like the "honey metaphor" where stirring a honey can move the specks even if the spoon doesn't touch them. But I'm not satisfied with such simplistic explanations.


So if I understood it right rotating object cause a force that push nearby objects in the direction of rotation. Is that right, or something else happens?


Is this a real force? So if I'm in the vicinity of a massive rotating object would I measure acceleration as I dragged around?


Does this effect caused by the mere fact of the rotation or does it caused by the gravitational anomalies? I mean that most rotating objects are not perfectly homogeneous rotating spheres.


Can the stress energy tensor deal with rotation? I seems it can encode energy density, linear momentum, pressure and shear stress but doesn't seem to be encode angular momentum, can it?



Answer



First I'll address your stress tensor question. Angular momentum is encoded in the following tensor: $$M^{\mu\nu\rho}=x^\nu T^{\mu\rho}-x^\rho T^{\mu\nu}$$ We then define the angular momentum tensor as $$J^{\mu\nu}=\int_\Sigma M^{0\mu\nu}$$ where the measure on the space-like hypersurface $\Sigma$ is understood. There are problems with this, however. This is because it is very difficult to distinguish between the energy-momentum of the gravitational field and the stuff in it. Take a perfect fluid, for instance. The stress tensor contains the metric explicitly! Thus the stress tensor for a perfect fluid also contains gravitational information. Because of this, calling $J^{\mu\nu}$ a tensor is not really right. (It should be noted that the energy-momentum of a gravitational field CAN be covariantly defined for an asymptotically flat manifold.)



Your other question is answered in chapter 7.5 of Einstein Gravity in a Nutshell by A. Zee (2013). I'll hit the key points. A Kerr spacetime is axisymmetric and thus has a Killing vector $\xi=\partial_\varphi$. There is a well-known theorem that states if $u$ is the tangent of a geodesic, then $\langle\xi,u\rangle$ is a conserved quantity. Define the momentum $p=mu$. Then $L=\langle\xi,p\rangle$ is a constant of motion, the angular momentum. The general axisymmetric metric is $$ds^2=g_{tt}dt^2+g_{rr}dr^2+g_{\theta\theta}d\theta^2+g_{\varphi\varphi}d\varphi^2+2g_{t\varphi}dtd\varphi$$ In coordinates, $L=p_\varphi=g_{\varphi t}p^t+g_{\varphi\varphi}p^\varphi$. In the $(-+++)$ convention I am using, $g_{\varphi\varphi}>0$. Positive angular momentum is when the $\phi$ coordinate increases with increasing proper time. Hence, angular momentum is $+p_\varphi$.


It is conventional to define $L=ml$. Then $l=g_{t\varphi}\dot{t}+g_{\varphi\varphi}\dot\varphi$. Go out to infinity and prepare a test particle with $l=0$. Drop it towards the black hole. The asymptotic nature of the field means that far away, $g_{t\varphi}\sim0$ and $g_{\varphi\varphi}\sim1$. Thus $l=0$ means $\dot\varphi\sim0$, which is to be expected.


Here's the kicker: angular momentum is conserved. Thus $l\equiv 0$ identically. But suppose we, in our lab at infinity, take a look at the test particle. We see an angular velocity $$\omega\equiv \frac{d\varphi}{dt}=\dot\varphi/\dot t=-\frac{g_{t\varphi}}{g_{\varphi\varphi}}$$ which can totally be nonzero!


There is no force, technically speaking. The particle is just following a geodesic, which happens to take it along a path with angular velocity!


EDIT: Found this shortly after posting. Suppose $(M,g)$ is an axisymmetric spacetime with angular Killing vector $\psi$. The angular momentum of spacetime is given by the Komar integral $$J=\frac{1}{16\pi}\int_{S^2_\infty}\star d\psi$$ over the 2-sphere at infinity. See page 466 of Straumann (2013) for a proof. Now choose a hypersurface $\Sigma$ such that $\psi$ is tangent. Then $$J=-\int_\Sigma T_{\mu\nu}\psi^\mu n^\nu$$ where $T_{\mu\nu}$ are the components of the stress tensor and $n$ is a unit normal to $\Sigma$. This is problem 11.6 in Wald (1984).


EDIT II: I'd like to say a few words about energy-momentum in GR. (See chapter 3.7 of Straumann (2013) for the very thorough [and highly technical] discussion.) From the nonlinearity of Einstein's equation we know that gravitons have a nasty tendency to couple to each other. This means that it is in general not possible to separate the stress tensor like $$T=T_\text{matter}+T_\text{gravity}$$ because gravitons are coupling to everything and making life difficult. (However, if the metric is sufficiently close to the Minkowski metric, we may define the stress tensor of gravity to be the second order expansion of the Einstein tensor. [Chap 10.3 in Weinberg (1972)])


The solution is the so-called ADM formalism. It allows for a covariant description of energy, momentum and angular momentum on an asymptotically flat spacetime. We use the notation $$\tau^\alpha=T^\alpha+t_\text{LL}^\alpha$$ where $T^\alpha$ are the stress 1-forms and $t_\text{LL}^\alpha$ are the Landau-Lifshitz 1-forms. We then define $$\star M^{\alpha\beta}=x^\alpha\star\tau^\beta-x^\beta\star\tau^\alpha$$ Then $$J^{\mu\nu}=\int_\Sigma\sqrt{-g}\star M^{\mu\nu}$$ is the ADM angular momentum and is conserved if the gravitational field falls off sufficiently fast at spacelike infinity. Note that $T^\alpha$ is constructed out of the stress tensor and $t_\text{LL}^\alpha$ out of the tetrad.


Where is noncommutativity in the state-effect formalism of quantum mechanics?


In quantum information theory, one can adopt the basic formalism where every system is given by an operator algebra, state preparation procedures correspond to linear functionals on that algebra (macroscopic systems preparing quantum systems), yes-no measurements correspond to projections in that algebra (quantum system affecting a macroscopic measurement device), and finally evaluating a state at a given projection is interpreted as the relative frequency of obtaining `yes' for the given projection.


More general observables are given by projection-valued measures on a set of measurement outcomes, or equivalently, self-adjoint operators. The need for an operator algebra (in the notes above, for example) is apparently that the spectral projections of a self-adjoint operator must be found in the observable algebra.


The trouble I have here is that it is not clear where, in the above formalism, one is to multiply linear operators representing observables? If an observable corresponds to a measurement, hence giving a `reading' on a macroscopic device (which is stipulated in the definition of measurement), how is it possible to perform a second measurement once a first has been made? (I'm thinking of a detector, here...like in Stern-Gerlach or double-slit.)


Does the need for noncommutativity come from an observed statistical uncertainty relation? Does one prepare a system in a specific way, say one of the beams of a Stern-Gerlach apparatus, and then make a measurement of two different quantities on the single prepared beam, and then try to observe a statistical uncertainty relation on the measured quantities? (i.e. does one look for an uncertainty relation in the standard deviations of the resulting measurements?) Is this how we'd determine that the observables assigned to the two measurements taken do not commute?


Note: "composed" Stern-Gerlach apparati and composition of quantum operations are not what I'm interested in. I want to know about noncommutativity of observables (which are viewed as measurements).


EDIT: It should be noted that a positive answer to the above question essentially protects uncertainty principle from weak measurement objections...


Thanks for your help in advance!


Further Edit:



Please tell me if the following is flawed. In classical mechanics, every observable is given by a real-valued function on the phase space. Each such function has a Fourier decomposition. In the case of a classical radiating atom under Maxwell's equations, it is predicted that the Fourier transform of the algebra of functions (observables) yields (and is isomorphic to) the convolution algebra of an additive abelian subgroup of the real numbers. However experimentally, the Rydberg-Ritz combination formula holds. Heisenberg drew a parallel between the convolution algebra of the group and that of the "Rydberg-Ritz rules" which yields a matrix algebra.


My question originally asked, essentially, how one is to get a noncommutative algebra by looking only at measurement results. This seems to be precisely what Heisenberg did.


Regarding the observables of this algebra (hermitian elements): at first glance it seems strange that Heisenberg assigns a noncommutative algebra to rules derived from spacing of frequencies of spectral lines, when Quantum Mechanics tells us to use measurement outcomes to label the eigenspaces of a hermitian operator in an operator algebra. I was under the impression that the spectral projections of the Hermitian operator, should generate the observable algebra. This sometimes holds in the commutative (classical) case by coincidence...since all the operators commute, a hermitian operator can determine a common eigenbasis for all observables, therefore the spectral projections generate the algebra. What an observable seems to do in the general (perhaps noncommutative matrix algebra) case is select an orthonormal basis with respect to which the matrices of the algebra are represented. If the observable is not invertible (e.g. a projection) this requires the additional choice of a basis for the kernel. Of course, everywhere I'm restricting my attention to finite dimensions.


The latter role for the Hermitian operator is not incompatible with the view that "measurement is final". We never need to compose two observables, but can effectively look at every operator in the algebra representing the system in terms of this basis. The noncommutativity came before this fact. (von Neumann) measurement, as orthogonal projection into eigenspaces of the given observable seems to make more sense from this viewpoint, as well.


Would a genuine physicist tell me if I'm off the mark here?




quantum field theory - Getting rid of double delta function in Feynman rules


[1] A very simple example of feynman rule for scalar fields.


After computing the diagram i have got the following:


$$ -i(2\pi)^4g^2\int d^4q \frac{i}{q^2 -m^2c^2}\delta^{(4)}(p_1 - p_3 -q) \delta^{(4)}(p_2 + q -p_4) $$


I'm a little confused about how the integral approached, it integrated over one delta function to get


$$ -ig^2\frac{1}{(p_4 - p_2)^2 -m^2c^2}(2\pi)^4\delta^{(4)}(p_1+p_2 - p_3 - p_4) $$


Am i allowed to do that? I mean I have $q$ in both delta functions. Can I just integrate over one of it? It doesn't sound right. What I'm missing here?



Answer



That looks correct to me. Consider the basic property of the delta functions $$ \int dx f(x) \delta(x-a) = f(a). $$ Nothing forbids $f(x)$ to be a composite function, for example $f(x) \equiv g(x)\delta(x-b)$, so $f(a) = g(a) \delta(a-b)$. Hence we get, $$ \int dx f(x) \delta(x-a) \equiv \int dx \, g(x)\delta(x-b) \delta(x-a) = g(a)\delta(a-b). $$



Friday, August 24, 2018

logical deduction - How to generalise Partition of the pie for N envy people?


There is well known task:



How to divide a pie between two people, so each will think that he got equal or more that the other one?



It has the following solution:




First person divide the pie on 2 equal (in his opinion) parts. Second one chooses the biggest one.



I know that it is possible to generalise in a way that N people will divide a pie on N pieces and each one will think that his part is not less than 1/N.


Is it possible to generalise this on N envy people? That means to divide a pie between N people in such a way that each one will think that his part is not less than part of any other man. In this case, when a man has got a part bigger than 1/N, he will still be following the rest of the division between the others - may be some one will get even bigger part (in this case the man will not calm down until the situation changes).




thermodynamics - Do conventional ovens heat by thermal conduction or radiation?


I am curious which energy transfer mechanism dominates in a conventional oven, thermal conduction or thermal radiation?


A naive guess would be that early on, when the food is still cold, the heat transfer rate would be higher for radiative than conductive transfer due to the dependence on $\Delta T^{4}$ for the former compared to $\Delta T$ for the latter. However, it is not clear to me if the oven elements/burners heat the air and the air transfers the heat through conduction or the oven walls radiate in infrared and heat through radiative transfer.


If we are heating a solid food (e.g., beef/meat) and we assume the air inside the oven is a constant, uniform temperature then we only have conductive and radiative heat processes, correct? Meaning, I need not worry about convection.


If these assumptions are correct, then what heat transfer mechanism will dominate and why?



Answer



Let's assume the oven walls radiate like perfect black body radiators and the air inside the oven is uniform throughout (ignore the thin layer near the cooler meat). Assume I want to cook a prime rib (chosen because it can be bought in a cut that has a cylindrical shape), which is just beef and fat. I can look up the thermal emissivity and thermal conductivity of beef to find:




  • $\epsilon$ ~ 0.74–0.78, depending on fat content; and

  • $\kappa$ ~ 0.504–0.561 W m-1 K-1, depending on fat content.


We know the heat transfer rate for radiative heating can be approximated by: $$ \dot{Q}_{rad} \approx \sigma \ \epsilon \ A \ \left( T_{oven}^{4} - T_{beef}^{4} \right) \tag{1} $$ where $\sigma$ is the Stefan-Boltzmann constant, $\epsilon$ is the radiative emissivity, $A$ is the surface area of the absorber, and $T_{j}$ is the temperature of the jth object.


Similarly, we know heat transfer rate for conductive heating can be approximated by: $$ \dot{Q}_{con} \approx \frac{ \kappa \ A \left( T_{oven} - T_{beef} \right) }{ L } \tag{2} $$ where $\kappa$ is the thermal conductivity, $A$ is the surface area of the absorber, $L$ is the thickness of the absorber, and again $T_{j}$ is the temperature of the jth object.


Let's assume we buy a prime rib that is ~1.5 ft (~0.46 m) in length and ~5 in (~0.13 m) in diameter and approximate it as a cylinder. Then the absorbing surface area and thickness would be $A$ ~ 0.42 m2 and $L$ ~ 0.13 m. If the meat starts at 32 oF (273.15 K) and the oven is at 350 oF (449.82 K), the rates from Equations 1 and 2 when the meat first enters the oven are:



  • $\dot{Q}_{rad}$ ~ 630–664 W; and

  • $\dot{Q}_{con}$ ~ 297–331 W.



Obviously this is a function of time since the $\Delta T$ will change with time. The constant factors for each are $\sim 1.8 \times 10^{-8}$ W K-4 for $\dot{Q}_{rad}$ and $\sim 1.8$ W K-1 for $\dot{Q}_{con}$. One can go ahead and plot the transfer rates (see figure below) and find that radiative transfer always dominates.


Radiative vs. Conductive Heating Rates



If these assumptions are correct, then what heat transfer mechanism will dominate and why?



So if we can truly ignore convection, then it appears that radiation dominates over conduction.


Entropy and the principle of least action


Is there any link between the law of maximum entropy and the principle of least action. Is it possible to derive one from the other ?




english - Longest *Sentence* With Only Repeating Character Pairs


BACKGROUND



I asked a question about words: Longest Word With *Only* Repeating Character Pairs


The goal was to find a word where every character pair was repeated at least once. This was vulnerable to dictionary searches and didn't require human creativity. In addition, every answer was of the format [string][same string][first letter of string]. For instance, ALFALFA is [ALF][ALF][A]. In fact, from my own quick and non-technical figurings, this format is the only possible format that can fit such a challenge. The answers found were interesting but it was this conclusion that I found more interesting.




CHALLENGE


Form a grammatically correct sentence in which every letter pair appears at least twice.


Clarifications: (The example below is invalid and is only intended to assist in clarification.)



  • Words can not be repeated but variations of a word do not count as a repetition. This includes plurals, possessives, contractions, etc. Uni the Unicorn's eating the unicorns!

  • Words count as a repetition if they match when converted to all uppercase and any non-letters are removed

  • Any non-letter (spaces, hyphens, apostrophes, etc.) should be stripped prior to analysis. The result will be just a string of letters. UnitheUnicornseatingtheunicorns


  • Capitalization does not matter. Convert them all to uppercase if that helps. UNITHEUNICORNSEATINGTHEUNICORNS




Disclosure: I don't have a solution yet.



Answer




Poorly named IT company launches their new product


The Stand - for all your system sensationalising and standardising needs. A systemic and systematic approach to system standardisation.



Sensational Lice Mice Stand sensationally systematically re-systems, sensationalised standardised standards, standardising sensationalising standard systematisation system systematisations, un-systematically re-standardised standardisation systems, unstandardised systematic and systemic.




The sensational Mice with the Lice have a new product the "Stand" that sensationally systematically does re-systemming of sensationalised standardised standards. As well as standardising the sensationalising of standard of the systematisations of systematisation systems, as well as un-systematically re-standardised standardisation systems, and the unstandardised systematic and systemic



Much longer sentences can be made using lists of things, it is probably possible the make a sentence using every word as a long enough list will eventually contain every pair of characters. Even those pairs that don't appear in words or appear in only single words can be formed from the letters of adjacent words.



classical mechanics - Moment of a force about a given axis (Torque) - Scalar or vectorial?

I am studying Statics and saw that: The moment of a force about a given axis (or Torque) is defined by the equation: $M_X = (\vec r \times \...